Πέμπτη 31 Ιουλίου 2014

$χ_1 <χ_2 < χ_3$

Έστω  οι πραγματικές ρίζες της εξίσωσης 
Να υπολογισθεί η τιμή της παράστασης .
USA AIME 2014

2 σχόλια:

  1. Αυτό το σχόλιο αφαιρέθηκε από τον συντάκτη.

    ΑπάντησηΔιαγραφή
  2. Έστω ${x_1} < {x_2} < {x_3}$ οι πραγματικές ρίζες της εξίσωσης $\sqrt {2014} {x^3} - 4029{x^2} + 2 = 0$. Να υπολογιστεί η παράσταση : ${x_2}({x_1} + {x_3})$.

    Λύση

    Επειδή $4029 = 2 \cdot 2014 + 1$ αν θέσουμε $\boxed{a = \sqrt {2014} }$ η εξίσωση θα μετασχηματιστεί στην $a{x^3} - (2{a^2} + 1){x^2} + 2 = 0$. Η εξίσωση ισοδυναμεί με : $a{x^3} - 2{a^2}{x^2} - {x^2} + 2 = 0 \Leftrightarrow a{x^3} - {x^2} - 2({a^2}{x^2} - 1) = 0$ . Εύκολα τώρα έχουμε :
    ${x^2}(ax - 1) - 2(ax - 1)(ax + 1) = 0 \Leftrightarrow $
    $\boxed{(ax - 1)({x^2} - 2ax - 2) = 0}$
    Η μια ρίζα της είναι προφανώς$\boxed{{r_0} = \frac{1}{a}}$ ενώ οι ρίζες του τριωνύμου είναι ετερόσημες $\boxed{{r_1} < 0 < {r_2}}$. Για τις ρίζες λοιπόν της αρχικής με τη δεδομένη διάταξη θα έχουμε : ${x_1} \equiv {r_1},{x_2} \equiv {r_0}\,\,\,\kappa \alpha \iota \,\,\,{x_3} \equiv {r_2}$ συνεπώς : ${x_2}({x_1} + {x_3}) = {r_0}({r_1} + {r_2}) = \dfrac{1}{a} \cdot 2a = 2$.

    Φραγκάκης Νίκος(Doloros) 2ο Λύκειο Ιεράπετρας

    ΑπάντησηΔιαγραφή